0
$\begingroup$

Let $$A = \Big\{(a_1,a_2,\dots)\ \Big|\ a_i\ge 0, \sum_{i=1}^\infty a_i=1\Big\},$$ $$v(x)=\sup\left(\bigg\{\sum_{i=1}^\infty a_ib_i\ \bigg|\ (a_i)_{i=1}^\infty,\, (b_i)_{i=1}^\infty \in A,\,\sup\limits_{i\in \mathbf N} a_ib_i =x\bigg\}\right).$$ Certainly $v$ is an increasing function. Is $v(x)$ finite for every $x$? Is it achievable? Does $v(x)\rightarrow 0$ as $x\rightarrow 0$?

$\endgroup$

2 Answers 2

1
$\begingroup$

We have the following inequality:

\begin{align*}\sum_{i =1}^\infty a_i b_i & \le \left(\sum_{i = 1}^\infty a_i^2 b_i\right)^{1/2} \left(\sum_{i = 1}^{\infty} b_i\right)^{1/2} \\ & \le \left(\sum_{i = 1}^\infty a_i a_i b_i\right)^{1/2} \\ & \le \left((\sup_i a_i b_i) \sum_{i =1}^\infty a_i\right)^{1/2}\\ & \le \sqrt{x}.\end{align*}

Hence $\nu(x) \to 0$ as $x \to 0$.

$\endgroup$
3
  • $\begingroup$ Thank you for the elegant solution. Would you mind I add a very slight alternative? $\endgroup$
    – Hans
    Oct 16, 2014 at 18:10
  • $\begingroup$ Of course not. Would you mind to explain me why you encounter such question? $\endgroup$
    – Yanqi QIU
    Oct 16, 2014 at 20:34
  • $\begingroup$ I was actually banging my head: why I did not think of this solution earlier... :-/ For the origin of the problem, you can take a look at my answer to my own question quant.stackexchange.com/questions/14994/…. $\endgroup$
    – Hans
    Oct 16, 2014 at 21:29
0
$\begingroup$

Inspired by the solution of Yanqi Qiu, here is a more symmetric derivative of it.

\begin{align} \Big( \sum_i a_ib_i\Big)^2 &= \Big(\sum_i (a_ib_i)^\frac{1}{2}a_i^\frac{1}{2}b_i^\frac{1}{2}\Big)^2 \\ &\le \sup\limits_i (a_ib_i)\Big(\sum_i a_i^\frac{1}{2}b_i^\frac{1}{2}\Big)^2 \\ &\le \sup\limits_i (a_ib_i)\sum_i a_i\sum_ib_i. \end{align}

$\endgroup$

Your Answer

By clicking “Post Your Answer”, you agree to our terms of service and acknowledge you have read our privacy policy.

Not the answer you're looking for? Browse other questions tagged or ask your own question.